Vous êtes sur la page 1sur 3

1. If r and s are positive integers, is r/s an integer?

(1) Every factor of s is also a factor of r.


(2) Every prime factor of s is also a prime factor of r.
Ans: A
2. If x is an integer, does x have a factor n such that 1 < n < x?

(1) x > 3!

(2) 15! + 2 x 15! + 15
Ans: B
(1) --> is more than some number (3!). may or may not be a prime. Not sufficient.

(2) --> can not be a prime. For instance
if , then is a multiple of
8, so not a prime. Same for all other numbers in this range: , where will definitely be a
multiple of (as weould be able to factor out out of ). Sufficient.
3. If n and t are positive integers, is n a factor of t ?

(1) n = 3^(n-2)
(2) t = 3^n
Ans: C
(1) n = 3^(n-2) --> n=3 (only integer solution for this equation), but we know nothing about t, so this statement is not sufficient.

(2) t = 3^n --> if n=1 then the answer will be YES but if n=2 then t=9 and the answer will be NO. Not sufficient.

(1)+(2) As n=3 then t=3^n=27 and the answer to the question will be YES as 3 is a factor of 27. Sufficient.
4. Let [[x]] represent the average of the greatest integer less than or equal to x and the least integer greater than or equal to x. Is 0 <=
|a| <= 1 ?
Hi Smita

1) [[x]] - x = a
2) 0 < [[a]] < 1
ANS: D
Let's try to understand the meaning of [[x]].
If x = 4.1, [[x]] = (4 + 5)/2 = 4.5
If x = 3.9, [[x]] = (3 + 4)/2 = 3.5

So, if x = Integer + d (where 'I' is its decimal part whereas 'd' is its decimal part),
then [[x]] = I + 0.5

Statement(1):
[[x]] - x = (I + 0.5) - (I + d) = 0.5 - d
|0.5 - d| will always be between 0 and 1, since d is between 0 and 0.999...
SUFFICIENT.

Statement(2):
0< [[a]] < 1
0 < I + d < 1
Since the decimal value is always between 0 and 1, the integral value of a = 0
So,
0 < a < 1
or
0 <= |a| <= 1
SUFFICIENT.
So the answer is D
600 LEVEL
WORK/RATE
1. Reserve tank 1 is capable of holding z gallons of water. Water is pumped into tank 1, which starts off empty at a rate of x gallons
per minute. Tank 1 simultaneously leaks water at a rate of y gallons per minute (x>y).The water that leaks out of tank 1 drips into
tank 2,which also starts off empty. If the total capacity of tank 2 is twice the number of gallons that remains in tank 1 after 1
minute, does tank 1 fill up before tank 2?

(1) zy < 2x^2-4xy+2y^2
(2) Total capacity of tank 2 is less than one half that of tank 1.
Ans: A
capacity of tank 2 is u = 2*(x-y)

time it takes for tank 2 to get filled is 2*(x-y)/y
time it takes for tank 1 to get filled is z/(x-y)

z/(x-y) - 2*(x-y)/y < 0? equivalent (since y > 0 , x-y > 0)

zy < 2*(x-y)^2 = 2*x^2-4*x*y + 2*y^2 (1) is sufficient -> time 1< time 2 -> tank 1 will get filled first
2. Danny and Ian are munching on a jar full of candies. Had Danny eaten alone it would have taken him 5 minutes to finish the
candies in the jar. Had Ian eaten alone it would have taken him 10 minutes to finish half the jar.
Since both of them are eating simultaneously, how long would it take them to empty the jar?

2.5 minutes

3 minutes

3 minutes and 20 seconds

6 minutes and 40 seconds

4 minutes
Ans: 4 mins
Danny's does in 5 minutes
Ian finishes half in 10 minutes i.e finishes the jar in 20 minutes
Therefore Combined rate will be

1/5+1/20=1/T

5/20= 1/T ==> T = 4 min
3. The third-place finisher of the Allen County hot dog eating contest, in which each contestant was given an equal amount of time
to eat as many hot dogs as possible, required an average of 15 seconds to consume each hot dog. How many hot dogs did the
winner eat?

(1) The winner consumed 24 more hot dogs than did the third-place finisher.

(2) The winner consumed hot dogs at double the rate of the third-place finisher.
Ans: C
1)

No hot dogs consumed by third placer=N3 = t *1/15
N3+24=t*r

insufficient.. we need to know the t or r

2)
N3= t*1/15
N1 = t*2/15
insuffcient

combined.

N1=2N3=N3+24

N3=24
N1=48
sufficient.
4. Alice, Barbara, and Cynia work on identical tasks at different constant rates. Alice, working alone, can complete the task in 21
hours. Is Alice's rate the slowest rate?

(1) Barbara, working alone, can complete the task in 14 hours, and Barbara and Cynia working together can complete the task in
approximately 86% of the time taken by Alice and Cynia working together to complete the task.

(2) Barbara and Cynia can complete the task in approximately 71% of the time taken for Alice and Barbara to complete the task.
Ans: A
We know that A takes 21 hours to complete the job alone. From the prompt we know zilch about how she compares to B or C.

Statement #1: Barbara, working alone, can complete the task in 14 hours, and Barbara and Cynia working together can complete
the task in approximately 86% of the time taken by Alice and Cynia working together to complete the task.
OK, so first of all, they tell us that B takes only 14 hours, so B definitely works faster than A.

Then we find out that B & C take less time than A & C, which means B & C works faster than A & C. Well, hmm. We already know,
from the first part of this statement, that B works faster than A. From casual inspection, it's not clear that this second statement
adds more information. We have to investigate it numerically, so we can involve that 86% figure, which is approximately 5/6.

A finishes in 21 hours, so her rate (job/hours) is 1/21. B's rate is 1/14. Let's say C takes c hours, so C's rate is 1/c. For
combinations of two people working together, we add rates.
Rate of B & C = 1/14 + 1/c
Rate of A & C = 1/21 + 1/c
B & C take only 5/6 the time of A & C, which means B & C have 6/5 the rate of A & C.

(1/14 + 1/c) = 6/5(1/21 + 1/c)

At this point, we have a single equation we could solve for c. This is GMAT DS. The trap of DS would be to go through all the steps
to find the numerical value of c. At this point, we see that we could solve for c. That's enough. If we knew c, we would know who
was the fastest and slowest, and we would be able to answer the prompt question. This statement, alone and by itself,
is sufficient.

Statement #2: Barbara and Cynia can complete the task in approximately 71% of the time taken for Alice and Barbara to complete
the task
Here, we have two unknowns, because we don't know C's rate or time, and we also don't know B's rate or time. Remember, B's rate
is something given in statement #1, so we can't use it here. We would be able to set up a single equation, but one equation with
two unknowns is not enough to solve, and we would still be left with questions. This statement, alone and by itself, isinsufficient.

Vous aimerez peut-être aussi